Đến nội dung

Hoang Tung 126 nội dung

Có 1000 mục bởi Hoang Tung 126 (Tìm giới hạn từ 29-04-2020)



Sắp theo                Sắp xếp  

#478766 giai phuong trinh $\left ( x-1 \right )^{2}=2-x...

Đã gửi bởi Hoang Tung 126 on 24-01-2014 - 16:12 trong Bất đẳng thức và cực trị

em thấy hình như thiếu TH do x chưa đươngầu 

Đầu tiên phải tìm ĐKXĐ nhưng quên




#478758 Đường thẳng Euler của tam giác DEF đi qua một điểm cố định

Đã gửi bởi Hoang Tung 126 on 24-01-2014 - 14:45 trong Hình học

Bài toán : Cho tam giác ABC có I là tâm đường tròn nội tiếp .Đường tròn tâm I tiếp xúc với BC,CA,AB tại D,E,F .

  CMR: Đường thẳng Euler của tam giác DEF đi qua một điểm cố định




#478755 tìm 2 số x,y là hai số nguyên thoã $x^{3}+x^{2}y+xy^...

Đã gửi bởi Hoang Tung 126 on 24-01-2014 - 14:40 trong Số học

tìm 2 số x,y là hai số nguyên thoã $x^{3}+x^{2}y+xy^{2}=4(x^{2}+y^{2}+xy+3)$

PT $< = > x(x^2+xy+y^2)=4(x^2+xy+y^2)+12< = > (x-4)(x^2+xy+y^2)=12=1.12=2.6=3.4$

Đến đây xét các TH là ra và lưu ý $x^2+xy+y^2\geq 0$




#478754 giai phuong trinh $\left ( x-1 \right )^{2}=2-x...

Đã gửi bởi Hoang Tung 126 on 24-01-2014 - 14:32 trong Bất đẳng thức và cực trị

giai phuong trinh $\left ( x-1 \right )^{2}=2-x\sqrt{x-\frac{1}{x}}$

PT $< = > (x-1)^2=2-x\sqrt{\frac{x^2-1}{x}}=2-\sqrt{x(x^2-1)}< = > x^2-2x+1=2-\sqrt{x.(x^2-1)}< = > x^2-2x-1+\sqrt{x(x^2-1)}=0< = > (x^2-2x-1)^2=x(x^2-1)< = > x^4-4x^3+2x^2+4x+1=x^3-x< = > x^4-5x^3+2x^2-5x+1=0$

Đến đây đơn giản vì nó là pt đối xứng




#478752 CMR : MD, NE, PF đồng quy .

Đã gửi bởi Hoang Tung 126 on 24-01-2014 - 14:28 trong Hình học

 Bài toán :Cho tam giác ABC nội tiếp đường tròn tâm O và H là điểm bất kì nằm trong tam giác .Nối AH, BH ,CH cắt đường tròn tâm O tại D ,E ,F .Gọi M ,N,P theo thứ tự là tâm đường tròn ngoại tiếp các tam giác BHC, AHB, AHC .

    CMR : MD, NF, PE đồng quy .

                                                                




#478748 a,b,c là 3 số dương thỏa a+b+c+1=4abc. CMR: $\frac{1}...

Đã gửi bởi Hoang Tung 126 on 24-01-2014 - 14:22 trong Bất đẳng thức - Cực trị

Cho a,b,c là 3 số dương thỏa a+b+c+1=4abc. CMR: $\frac{1}{a^{4}+b+c}+\frac{1}{b^{4}+c+a}+\frac{1}{c^{4}+a+b}\leq \frac{3}{a+b+c}$

Ta có :$a+b+c+1=4abc= > \frac{1}{ab}+\frac{1}{bc}+\frac{1}{ac}+\frac{1}{abc}=4$

Đặt $\frac{1}{a}=x,\frac{1}{b}=y,\frac{1}{c}=z= > xy+yz+xz+xyz=4$

BĐT $< = > \sum \frac{1}{a^4+b+c}=\sum \frac{\frac{1}{a^2}+b+c}{(a^4+b+c)(\frac{1}{a^2}+b+c)}\leq \sum \frac{\frac{1}{a^2}+b+c}{(a+b+c)^2}=\frac{\sum \frac{1}{a^2}+2\sum a}{(\sum a)^2}\leq \frac{3}{\sum a}< = 3\sum a\geq \sum \frac{1}{a^2}+2\sum a< = > \sum a\geq \sum \frac{1}{a^2}< = > a^2b^2c^2(\sum a)\geq \sum a^2b^2=(\sum ab)^2-2abc\sum a< = > abc\left [ abc(\sum a)+2 \right ]\geq (\sum ab)^2$




#478745 $A=\sum \frac{(a+b)c}{(a+b)^{2}+c^...

Đã gửi bởi Hoang Tung 126 on 24-01-2014 - 14:11 trong Bất đẳng thức và cực trị

Cho a,b,c là các số dương. CMR:

$A=\sum \frac{(a+b)c}{(a+b)^{2}+c^{2}}\leq \frac{6}{5}$

Cách khác .Chuẩn hoá $a+b+c=3$.Do đó 

BDT $< = > \sum \frac{c(3-c)}{c^2+(3-c)^2}\leq \frac{6}{5}< = > \sum \frac{1}{2c^2-6c+9}\leq \frac{3}{5}$

Mà $\frac{1}{2c^2-6c+9}\leq \frac{2c+3}{25}< = > 2c^3+1\geq 3c^2$(Luôn đúng theo AM-GM cho 2 số)

 $= > \sum \frac{1}{2c^2-6c+9}\leq \sum \frac{2c+3}{25}=\frac{2\sum c+9}{25}=\frac{2.3+9}{25}=\frac{15}{25}=\frac{3}{5}$




#478744 $P=\left ( x^4+y^4+z^4 \right )\left ( \dfrac{1...

Đã gửi bởi Hoang Tung 126 on 24-01-2014 - 13:54 trong Bất đẳng thức và cực trị

Cho $x,y,z$ là những số thực dương thỏa mãn $x+y\leq z$. Tìm giá trị nhỏ nhất của biểu thức $P=\left ( x^4+y^4+z^4 \right )\left ( \dfrac{1}{x^4} +\dfrac{1}{y^4} +\dfrac{1}{z^4} \right )$

Ta có :$P=(x^4+y^4+z^4)(\frac{1}{x^4}+\frac{1}{y^4}+\frac{1}{z^4})=3+(\frac{x^4}{y^4}+\frac{y^4}{x^4})+z^4(\frac{1}{x^4}+\frac{1}{y^4})+\frac{x^4+y^4}{z^4}\geq 3+2\sqrt{\frac{x^4}{y^4}.\frac{y^4}{x^4}}+\frac{2z^4}{x^2y^2}+\frac{(x+y)^4}{8z^4}\geq 5+\frac{2z^4}{\frac{(x+y)^4}{16}}+\frac{(x+y)^4}{8z^4}=5+\frac{32z^4}{(x+y)^4}+\frac{(x+y)^4}{8z^4}=5+(\frac{(x+y)^4}{8z^4}+\frac{z^4}{8(x+y)^4})+\frac{255z^4}{8(x+y)^4}\geq 5+2\sqrt{\frac{1}{64}}+\frac{255z^4}{8.z^4}=\frac{297}{8}= > P\geq \frac{297}{8}$

(Do áp dụng bdt Cosi và $x+y\leq z$)

  Dấu = xảy ra tại $x=y=\frac{z}{2}$




#478221 Tìm Min của $P=\dfrac{a^4}{(b-1)^3}+\dfrac...

Đã gửi bởi Hoang Tung 126 on 20-01-2014 - 17:21 trong Bất đẳng thức và cực trị

1. Cho $\begin{cases}a>1;b>1\\a+b\le 4\end{cases}$. Tìm Min của $P=\dfrac{a^4}{(b-1)^3}+\dfrac{b^4}{(a-1)^3}$

2. Cho $x,y,z>0$ thỏa mãn $x+y+z=3$ . Chứng minh:

$$\sum\frac{x(y+z)}{4-yz}\ge 2xyz$$

Bài 1: Theo AM-GM có :$\frac{a^4}{(b-1)^3}+16(b-1)+16(b-1)+16(b-1)\geq 4\sqrt[4]{16^3.a^4}=32a$

                                      $\frac{b^4}{(a-1)^3}+16(a-1)+16(a-1)+16(a-1)\geq 32b$

Cộng theo vế $= > P\geq 96-16(a+b)\geq 96-16.4=32$




#477903 bất đẳng thức lớp 9

Đã gửi bởi Hoang Tung 126 on 18-01-2014 - 19:28 trong Bất đẳng thức và cực trị

Bài 2: BĐT $< = > (\frac{a}{b}-\frac{a+2012}{b+2012})+(\frac{b}{c}-\frac{b+2012}{c+2012})+(\frac{c}{a}-\frac{c+2012}{a+2012})\geq 0< = > \frac{a-b}{b(b+2012)}+\frac{b-c}{c(c+2012)}+\frac{c-a}{a(a+2012)}\geq 0< = > \frac{a-b}{b(b+2012)}+\frac{b-a+a-c}{c(c+2012)}+\frac{c-a}{a(a+2012)}\geq 0< = > (a-b)(\frac{1}{b^2+2012b}-\frac{1}{c^2+2012c})+(a-c)(\frac{1}{c^2+2012c}-\frac{1}{a^2+2012a})\geq 0$

Đến đây là xong




#477900 bất đẳng thức lớp 9

Đã gửi bởi Hoang Tung 126 on 18-01-2014 - 19:22 trong Bất đẳng thức và cực trị

Bài 1: Ta có :$\frac{1}{ab}+\frac{1}{bc}=\frac{1}{b}(\frac{1}{a}+\frac{1}{c})\geq \frac{1}{b}.\frac{4}{a+c}=\frac{4}{b(a+c)}\geq \frac{4}{\frac{(a+b+c)^2}{4}}=\frac{16}{(a+b+c)^2}=1$




#477857 $\sum \frac{x}{x^2+yz}\leq \dfra...

Đã gửi bởi Hoang Tung 126 on 18-01-2014 - 15:25 trong Bất đẳng thức và cực trị

Ta có :$P=\sum \frac{x}{x^2+yz}\leq \sum \frac{x}{2x\sqrt{yz}}=\frac{1}{2}\sum \frac{1}{\sqrt{yz}}\leq \frac{1}{2}(\sum \frac{1}{x})=\frac{\sum xy}{2xyz}\leq \frac{\sum x^2}{2xyz}=\frac{1}{2}$




#477856 $11^a+3^a=2.7^a$

Đã gửi bởi Hoang Tung 126 on 18-01-2014 - 15:23 trong Phương trình - hệ phương trình - bất phương trình

Cho $a$ là số thực .Giải phương trình sau :

                         $11^a+3^a=2.7^a$




#477855 $P=\dfrac{x^3+y^3+16z^3}{(x+y+z)^3}$

Đã gửi bởi Hoang Tung 126 on 18-01-2014 - 15:22 trong Bất đẳng thức và cực trị

cho x,y,z $\geq 0$ và x+y+z>0.Tìm Min P:

 

$P=\dfrac{x^3+y^3+16z^3}{(x+y+z)^3}$

Theo bdt Holer có :$(x+y+z)^3=(x.1.1+y.1.1+\sqrt[3]{16}z.1.\frac{1}{\sqrt[3]{16}})^3\leq (x^3+y^3+16z^3)(1^3+1^3+1^3)(1^3+1^3+\frac{1}{16})= > P=\frac{x^3+y^3+16z^3}{(x+y+z)^3}\geq \frac{1}{3.\frac{32}{16}}=\frac{16}{96}$




#477815 Bài 1: $\left\{\begin{matrix} x + y + xy =...

Đã gửi bởi Hoang Tung 126 on 18-01-2014 - 08:02 trong Phương trình - hệ phương trình - bất phương trình

Bài 1: Đặt $x+y=a,xy=b= > a+b=2m+1,ab=m^2+m$

$= > a,b$ là 2 nghiệm của pt $X^2-(2m+1)X+m^2+m=0$

Để hệ có nghiệm duiy nhất thì pt này phải có nghiệm duy nhất hay $\Delta =(2m+1)^2-4(m^2+m)=0< = > 1=0$(vô lý)




#477814 Tìm m để BPT có nghiệm $\sqrt{x}+\sqrt[4]{x...

Đã gửi bởi Hoang Tung 126 on 18-01-2014 - 07:59 trong Phương trình - hệ phương trình - bất phương trình

Theo Bunhiacopxki có :$\sqrt{x}+\sqrt{2-x}\leq \sqrt{2(x+2-x)}=2$

                                     $\sqrt[4]{x}+\sqrt[4]{2-x}\leq \sqrt[4]{8(x+2-x)}=\sqrt[4]{16}=2$

Cộng theo vế $= > \sqrt{x}+\sqrt[4]{x}+\sqrt{2-x}+\sqrt[4]{2-x}\leq 2+2=4$

 Vậy để pt có nghiệm thì $m\geq 4$




#477813 $A=\frac{1}{2-cosA}+\frac{1}...

Đã gửi bởi Hoang Tung 126 on 18-01-2014 - 07:52 trong Bất đẳng thức - Cực trị

Chứng minh tổng các x2/(2x2 + yz) <= 1 như thế nào thế bạn

BĐT $< = > \sum \frac{yz}{2x^2+yz}\geq 1$ và theo Bunhia có :$\sum \frac{yz}{yz+2x^2}=\sum \frac{y^2z^2}{y^2z^2+2x^2yz}\geq \frac{(\sum yz)^2}{\sum y^2z^2+2xyz(\sum x)}=1$




#477812 .CMR: $\frac{AB+BC+CA}{4}<R+r$

Đã gửi bởi Hoang Tung 126 on 18-01-2014 - 07:46 trong Bất đẳng thức và cực trị

Cho $\Delta ABC$ có R là bán kính đường tròn ngoại tiếp, r là bán kính đường tròn nội tiếp .CMR: $\frac{AB+BC+CA}{4}<R+r$

Đặt $AB=c,BC=a,CA=b$

Ta có :$R=\frac{abc}{4S}=\frac{abc}{4\sqrt{p(p-a)(p-b)(p-c)}},r=\frac{2S}{a+b+c}=\frac{2\sqrt{p(p-a)(p-b)(p-c)}}{a+b+c}$

Đặt $p-a=x,p-b=y,p-c=z= > a=y+z,b=x+z,c=x+y,a+b+c=2(x+y+z)$

BĐT $< = > \frac{\sum x}{2}< \frac{(x+y)(y+z)(x+z)}{4\sqrt{xyz(\sum x)}}+\frac{\sqrt{xyz(\sum x)}}{\sum x}< = > 2(\sum x)^2\sqrt{xyz(\sum x)}\leq (x+y)(y+z)(x+z)+4xyz(\sum x)$

Nhưng bđt này luôn đúng do theo AM-GM có :

  $(x+y)(y+z)(x+z)(\sum x)+4xyz(\sum x)=(\sum x)((x+y)(y+z)(x+z)+4xyz)> (\sum x)((x+y)(y+z)(x+z)+xyz)=(\sum x)(\sum x)(\sum xy)=(\sum x)^2(\sum xy)\geq (\sum x)^2\sqrt{3xyz(\sum x)}=(\sum x)^2.\frac{3}{\sqrt{3}}\sqrt{xyz(\sum x)}> (\sum x)^2.\frac{2}{\sqrt{3}}\sqrt{xyz(\sum x)}$




#477742 $\frac{3}{\sqrt{3x+1}+2}\ge...

Đã gửi bởi Hoang Tung 126 on 17-01-2014 - 20:48 trong Phương trình - hệ phương trình - bất phương trình

PT $< = > 3\geq (2x+1)\sqrt{3x+1}+4x+2< = > 1-4x\geq (2x+1)\sqrt{3x+1}< = > (1-4x)^2\geq (3x+1)(2x+1)^2$




#477738 cho a,b,c,d là các số thực thoả mãn

Đã gửi bởi Hoang Tung 126 on 17-01-2014 - 20:44 trong Bất đẳng thức và cực trị

cho a,b,c,d là các số thực thoả mãn :

$\left\{\begin{matrix} a+b+c+d=7\\ \sum a^{2}=13 \end{matrix}\right.$

tìm max, min của a,b,c,d 

Áp dụng bđt Cosi có :$a^2+b^2+c^2\geq \frac{(a+b+c)^2}{3}< = > 13-d^2\geq \frac{(7-d)^2}{3}< = > 39-3d^2\geq d^2-14d+49< = > 4d^2-14d+10\leq 0< = > (d-1)(2d-5)\leq 0< = > 1\leq d\leq \frac{5}{2}$

Lập luận tương tự cho $a,b,c$




#477736 . Chứng minh bất đẳng thức sau: $a+b+c-abc\leq 4$

Đã gửi bởi Hoang Tung 126 on 17-01-2014 - 20:41 trong Bất đẳng thức và cực trị

Bạn ơi, thế cứ biết nhiều bất đẳng thức hay là làm được nhiều bài tập à!

ừ tùy vào từng bài thôi không phải bài này cũng dùng được đâu




#477671 $\sum \frac{a(b+c)}{b^2+bc+c^2}\geq 2...

Đã gửi bởi Hoang Tung 126 on 17-01-2014 - 14:06 trong Bất đẳng thức - Cực trị

Ta có :$(\sum \frac{a(b+c)}{b^2+bc+c^2})(\sum \frac{a(b^2+bc+c^2)}{b+c})\geq (\sum a)^2= > \sum \frac{a(b+c)}{b^2+bc+c^2}\geq \frac{(\sum a)^2}{\sum \frac{a(b^2+bc+c^2)}{b+c}}\geq 2< = > (\sum a)^2\geq 2\sum \frac{a(b^2+bc+c^2)}{b+c}=2\sum \frac{a(b+c)^2-abc}{b+c}< = > (\sum a)^2+2abc(\sum \frac{1}{b+c})\geq 4\sum ab< = > 2\sum a^2+2abc\sum \frac{1}{b+c}\geq (\sum a)^2< = > 2\sum a(a+\frac{bc}{b+c})\geq (\sum a)^2< = > 2\sum a.\frac{ab+bc+ac}{b+c}\geq (\sum a)^2< = > \sum \frac{a}{b+c}\geq \frac{(\sum a)^2}{2\sum ab}$(Luôn đúng theo BĐT Bunhiacopxki)




#477669 $P=\frac{x^2y}{z^3}+\frac{y^2z}...

Đã gửi bởi Hoang Tung 126 on 17-01-2014 - 13:53 trong Bất đẳng thức và cực trị

Cho x,y,z>0

Tìm giá trị nhỏ nhất biểu thức :

 

$P=\frac{x^2y}{z^3}+\frac{y^2z}{x^3}+\frac{z^2x}{y^3}+\frac{4xyz}{xy^2+yz^2+zx^2}$

 

P/s: không biết có ai có cách hay hơn cho bài này không. :icon10:  >:)  :icon12:

BĐT $< = > (\frac{x}{z})^2.\frac{y}{z}+(\frac{y}{x})^2.\frac{z}{x}+(\frac{z}{y})^2.\frac{x}{y}+\frac{4}{\frac{y}{z}+\frac{z}{x}+\frac{x}{y}}$

Đặt $\frac{x}{z}=a,\frac{y}{x}=b,\frac{z}{y}=c= > abc=1$

$= > A=\frac{a^2}{c}+\frac{b^2}{a}+\frac{c^2}{b}+\frac{4}{ab+bc+ac}\geq a+b+c+\frac{4}{ab+bc+ac}=\frac{4(\sum a)}{9}+\frac{4\sum a}{9}+\frac{4}{\sum ab}+\frac{\sum a}{9}\geq 3\sqrt[3]{\frac{16(\sum a)^2}{81\sum ab}}+\frac{3\sqrt[3]{abc}}{9}\geq 3\sqrt[3]{\frac{16.3\sum ab}{81\sum ab}}+\frac{1}{3}=\frac{13}{3}$




#477668 $\sum \frac{a}{\sqrt{b^{3}+...

Đã gửi bởi Hoang Tung 126 on 17-01-2014 - 13:47 trong Bất đẳng thức và cực trị

Theo AM-GM có :$\sum \frac{a}{\sqrt{b^3+1}}=\sum \frac{a}{\sqrt{(b+1)(b^2-b+1)}}\geq \sum \frac{a}{\frac{b+1+b^2-b+1}{2}}=\sum \frac{2a}{b^2+2}=2\sum \frac{a}{b^2+2}=\sum \frac{a(2+b^2)-ab^2}{b^2+2}=\sum a-\sum \frac{ab^2}{b^2+2}=6-\sum \frac{ab^2}{\frac{b^2}{2}+\frac{b^2}{2}+2}\geq 6-\sum \frac{ab^2}{3\sqrt[3]{\frac{b^4}{2}}}=6-\frac{\sqrt[3]{2}}{3}\sum \frac{ab^2}{b\sqrt[3]{b}}=6-\frac{\sqrt[3]{2}}{3}\sum a\sqrt[3]{b^2}=6-\frac{1}{3}\sum a\sqrt[3]{2.b.b}\geq 6-\frac{1}{9}.a(2b+2)=\frac{16}{3}-\frac{2\sum ab}{9}\geq \frac{16}{3}-\frac{2}{9}.\frac{(\sum a)^2}{3}=2$(ĐPCM)




#477568 CMR : $\frac{1}{5}+\frac{1}...

Đã gửi bởi Hoang Tung 126 on 16-01-2014 - 17:52 trong Bất đẳng thức và cực trị

CMR : $\frac{1}{5}+\frac{1}{13}+...+\frac{1}{n^2+(n+1)^2} < \frac{9}{20}$

Theo AM-GM có :$\frac{1}{k^2+(k+1)^2}< \frac{1}{2k(k+1)}$

$= > P=\frac{1}{5}+\frac{1}{13}+...+\frac{1}{n^2+(n+1)^2}< \frac{1}{2}(\frac{1}{1}-\frac{1}{2}+\frac{1}{2}-\frac{1}{3}+...-\frac{1}{(n+1)})=\frac{1}{2}(1-\frac{1}{n+1})< \frac{9}{20}$